LSAT and Law School Admissions Forum

Get expert LSAT preparation and law school admissions advice from PowerScore Test Preparation.

User avatar
 Dave Killoran
PowerScore Staff
  • PowerScore Staff
  • Posts: 5853
  • Joined: Mar 25, 2011
|
#22831
Complete Question Explanation

WeakenX. The correct answer choice is (D)

This is an excellent question, and a perfect example of how "older" LSAT questions are still entirely relevant to today's LSAT.

In this stimulus, the author tells us that although city taxes subsidize Greenville public bussing, the buses benefit outside commuters as well, so some city councilors suggest increasing fares to cover the cost of the service. However, some city councillors argue for raising the fares, on the basis that "taxes should be used primarily to benefit the people who pay them."

The question which follows the stimulus is a Weaken Except question, which means that the four incorrect answer choices will weaken the councillors' argument, and the correct answer choice will not weaken it. The first and most obvious line of attack would be to find and eliminate answers that show that the outside commuters do indeed pay taxes (and indeed several answers play on this approach) or that such a plan would have general negatives that would hurt the city on the whole. In other words, eliminate any answer that hurts the argument of the city councillors, and look for an answer that is irrelevant or strengthens the argument.

Answer choice (A): Because businesses in the city are also taxpayers, this answer shows that that raising the fares would hurt a group of taxpayers. In addition, there would be a general loss of benefit to the citizens of the city if businesses left. Thus, this answer choice weakens the argument and is incorrect.

Answer choice (B): This answer choice also weakens the argument that fares should be increased, by pointing out another detrimental result associated with the councillors' suggested plan. Thus, this answer choice weakens the argument and is incorrect.

Answer choice (C): This answer choice at first appears irrelevant because the low income residents don't pay taxes, and thus one might initially think that this answer is correct. However, the second half of the answer shows that "all city councilors agree" that these residents should be able to use the buses without disadvantage. Thus, if adopted, this answer would place the "some city councillors" from the stimulus in a contradictory position (as far as their stated goals here for low income residents to take advantage of the services vs the results of their plan which disadvantages low income residents), and that would weaken the argument. This is a very tricky and very well-placed answer (just before the correct answer), evocative of recent Logical Reasoning questions.

Answer choice (D): This is the correct answer choice. This choice fails to weaken the argument advanced in the stimulus, because voter preference about increasing local taxes is irrelevant—there is no tax increase even being suggested! The suggestion is to raise bus fares. Additionally, the opinions ("preferences") in this answer do not attack the facts cited above. Since this answer choice has no effect on the strength of the councillors' argument, this is the correct answer choice to this Weaken Except question.

Answer choice (E): This answer choice weakens the argument because it shows that those "outsiders" who commute into Greenville do in fact pay taxes to cover the services they use, and thus they would qualify as as people who should benefit from them. Note how carefully the stimulus worded the issues: they never used "residents," they simply used "people who pay them (taxes)." This answer then shows that the commuters are actually taxpayers, and consequently the city taxes would be used to benefit those who pay them. As this is the very basis of the councillors' objection to the subsidy, this answer would undermine their objection.
 satipod
  • Posts: 11
  • Joined: Apr 26, 2012
|
#22411
Hello!

I was inclined to pick C because it concerns people in Greenville who are exempt from taxes whereas the stimulus is talking about benefiting tax paying residents. Maybe I got too caught up with the wording there?

Thank you much!
 Steve Stein
PowerScore Staff
  • PowerScore Staff
  • Posts: 1153
  • Joined: Apr 11, 2011
|
#22412
Hi,

Regarding question #19: as you pointed out, this is a Weaken Except question, which means that the four incorrect answer choices will weaken the argument that those fares should be raised.

Answer choice C provides that everyone on the Council agrees that the low income residents should be allowed to take advantage of city services, and the fare increase would be disadvantageous to those residents--this is a point that would stack up against the fare increase plan.

Let me know whether that clears that one up--thanks!

~Steve
 satipod
  • Posts: 11
  • Joined: Apr 26, 2012
|
#22414
Thanks so much, that helps a lot. The reasoning #19 makes sense now! Gracias!
 ob00x7
  • Posts: 5
  • Joined: Apr 11, 2013
|
#22415
Hey I know its been a while since anyone has posted on this but i have a question about q19 as well. I understand that it is a weaken except question and i can understand that D does no weaken the argument. But i am having trouble seeing how E does. We know (1) that many people outside of the city use public transit to commute to work (2) some councillors want to raise fares because only people who pay the taxes should benefit from the service. As far as E weakening the argument it says that people who work in greensville and earn wages above the minimum all pay 5% tax. Even if this were true how do we know that the people who commute from outside the city that ride the bus earn above the minimum. They could drive use some other means of transportation, which would exclude them from E and therefore not weaken the argument. Maybe i am seeing something wrong or not seeing something but any help would be greatly appreciated and thank you in advance and happy new years.
 Nikki Siclunov
PowerScore Staff
  • PowerScore Staff
  • Posts: 1362
  • Joined: Aug 02, 2011
|
#22416
Hey ob00x7,

Thanks for your question. Read answer choice (E) carefully:
People who work in Greenville and earn wages above the nationally mandated minimum all pay the city wage tax of 5 percent.
Basically, as long as you make above-minimum wages and work in Greenville (regardless of where you live), you pay a city tax. So, the fact that non-resident commuters benefit from the subsidized bus fares should not be a problem: those guys pay taxes just like the city residents do. In other words, answer choice (E) makes it more likely that city taxes are being used to benefit the people who pay them, because everyone who benefits from the low fares pays the tax that makes such low fares possible (ok, everyone except for those making minimum wage). This removes the main objection of the city councillors, weakening their proposal to raise the bus fares.

Does this help? Let me know!
 ob00x7
  • Posts: 5
  • Joined: Apr 11, 2013
|
#22417
Thank you so much NIkki that was very helpful. This was one I was having trouble with early in my lsat studies. I went back to go over all the questions i did and this one just wasnt making sense. Thanks.
 yrivers
  • Posts: 68
  • Joined: Mar 15, 2017
|
#34796
Hi,

This question really got me and I'm having trouble after reading the explanation.

Specifically, answer C states "...make them EXEMPT FROM CITY TAXES...." (caps added). This phrase made me immediately choose this answer because these folks became irrelevant to the argument. They are not paying taxes, so they are not relevant to the "benefit the people who pay them (taxes)" in the argument. Can you explain why this is wrong?

As follow-up to the explanation above, how do we know that the folks commuting to Greenville and making money are "earn[ing] wages above the nationally mandated minimum"? That's the piece I didn't understand.

Thanks,
Yaesul
 Kristina Moen
PowerScore Staff
  • PowerScore Staff
  • Posts: 230
  • Joined: Nov 17, 2016
|
#34941
Hi yrivers,

The argument by the councilors is this:
"Some city councilors argue that city taxes should be used primarily to benefit the people who pay them..." (emphasis added)

So to weaken their argument, you'd want to provide a reason that fare subsidies DO benefit the people who pay taxes, even though most people who use come from outside city limits. You could also provide another reason why bus fares should NOT be raised. Any answer choice that weakens the argument (even if it's just a little bit!) is an incorrect answer. Remember that with a Weaken question, you are not looking for an answer choice that KILLS the argument, but one that weakens it. You want something that makes you go "Huh... all is not what it seems!" (And of course, with this question stem that means it's the incorrect answer choice)

Answer choice (E) makes it so that those commuters are paying the city taxes, and thus weakens the argument.

Answer choice (C) weakens the argument because of the last part of the answer. I will underline it:
"...and all city councilors agree that these residents should be able to take advantage of city-run services." This weakens the argument by providing a reason why bus fares should not be raised.
 martinbeslu
  • Posts: 49
  • Joined: Aug 09, 2017
|
#42452
Would answer choice C be considered an alternate cause for the given effect? Is that why it weakens the argument?

Answer choice C does provide a reason why bus fares should not be raised but I still don’t see how that matters in relation to the city councillor’s argument. I thought the answer choices were supposed to attack the logical flaws in the argument, not attack the premises given in the stimulus by saying to disregard what we just read previously. Is this one of those rare LSAT questions that attacks the premises instead of the conclusion?

Argument:
city taxes should primarily benefit the people who pay them —> raise bus fares

Answer choice C:
people that don’t pay taxes should benefit —> don’t raise bus fares

Get the most out of your LSAT Prep Plus subscription.

Analyze and track your performance with our Testing and Analytics Package.